Help understanding the Laurent series












0












$begingroup$


I just wanna see if I have some of the fundamentals nailed in terms of understanding.



Say we have some complex valued function defined as



$$f(z)=tfrac{1}{z(z-1)}$$



and we want to evaluate it's laurent series centred at $z_0=0$ on the annulus $0<|z|<1$.



Or take the function



$$f(z)=tfrac{1}{(z-1)(z+1)}$$



this time taking it's Laurent series centred at $z_0=1$ on the annulus $0<|z-1|<2$



I have a few questions regarding said laurent series:



i) my first question is just on the general situation weve got here . I believe in the first example I mentioned that we have a function which has two singularities, a simple pole at zero and another at 1. Then we have defined a region on which to calculate the laurent series that stops us from hitting the singularities by only calculating at a distance greater to one and less than the other . similarly we have in my example a complex valued function defined on some region but this time it's singularities are +-1 . and we want to centre our expansion at 1 so no we define our region in such a way that the distance between 1( which we'll never hit because we require the distance between it and any point to be greater than zero ) and any other point is less than 2 so we know have a circle that is centred at one and has one side touching 3 and the other -1 (it's singularity) but never touches either.Is this correct ?



ii)a. when actually calculating the series mentioned in my first series I know we evaluate in a region valid for $|z|<1$ in the following way :



$$tfrac{1}{z(z-1)}=tfrac{-1}{z}tfrac{1}{1-z}=tfrac{-1}{z} Sigma_{n=0}z^n$$ which is valid for $|z|<1/|c|$ c in this case being the coeffiecient of z in in the secon equality above.



My question however is this, evaluating for $|z|<1$ is the same as evaluating for $0<|z|<1$ in this case right ? , because the series i found is valid below 1 up until 0 which it is undefined, at right ?










share|cite|improve this question











$endgroup$












  • $begingroup$
    Your typography is a little difficult for me to read, but at least for your first question, I think you have the right idea. The region you define the Laurent series on won't contain a singularity, but the singularities can be on the boundary of the region (in fact, regions of convergence for Laurent series are either disks or annuli, depending on the singularities).
    $endgroup$
    – Clayton
    Nov 30 '18 at 18:55






  • 1




    $begingroup$
    We actually don't choose the region. The Laurent series itself does that. If you take the Laurent series around a pole, it's radius of convergence will be the distance to the next closest place the function is not analytic.Your final example is true everywhere that the expressions are defined. $$frac 1{1-z} = sum z^n$$ is true where the series converges, which includes $|z| < 1$ and excludes $|z| > 1$. But $$frac{-1}zfrac 1{1-z}=frac{-1}z sum z^n$$ also requires $frac 1z$ to exist, so excludes $z = 0$.
    $endgroup$
    – Paul Sinclair
    Dec 1 '18 at 5:50
















0












$begingroup$


I just wanna see if I have some of the fundamentals nailed in terms of understanding.



Say we have some complex valued function defined as



$$f(z)=tfrac{1}{z(z-1)}$$



and we want to evaluate it's laurent series centred at $z_0=0$ on the annulus $0<|z|<1$.



Or take the function



$$f(z)=tfrac{1}{(z-1)(z+1)}$$



this time taking it's Laurent series centred at $z_0=1$ on the annulus $0<|z-1|<2$



I have a few questions regarding said laurent series:



i) my first question is just on the general situation weve got here . I believe in the first example I mentioned that we have a function which has two singularities, a simple pole at zero and another at 1. Then we have defined a region on which to calculate the laurent series that stops us from hitting the singularities by only calculating at a distance greater to one and less than the other . similarly we have in my example a complex valued function defined on some region but this time it's singularities are +-1 . and we want to centre our expansion at 1 so no we define our region in such a way that the distance between 1( which we'll never hit because we require the distance between it and any point to be greater than zero ) and any other point is less than 2 so we know have a circle that is centred at one and has one side touching 3 and the other -1 (it's singularity) but never touches either.Is this correct ?



ii)a. when actually calculating the series mentioned in my first series I know we evaluate in a region valid for $|z|<1$ in the following way :



$$tfrac{1}{z(z-1)}=tfrac{-1}{z}tfrac{1}{1-z}=tfrac{-1}{z} Sigma_{n=0}z^n$$ which is valid for $|z|<1/|c|$ c in this case being the coeffiecient of z in in the secon equality above.



My question however is this, evaluating for $|z|<1$ is the same as evaluating for $0<|z|<1$ in this case right ? , because the series i found is valid below 1 up until 0 which it is undefined, at right ?










share|cite|improve this question











$endgroup$












  • $begingroup$
    Your typography is a little difficult for me to read, but at least for your first question, I think you have the right idea. The region you define the Laurent series on won't contain a singularity, but the singularities can be on the boundary of the region (in fact, regions of convergence for Laurent series are either disks or annuli, depending on the singularities).
    $endgroup$
    – Clayton
    Nov 30 '18 at 18:55






  • 1




    $begingroup$
    We actually don't choose the region. The Laurent series itself does that. If you take the Laurent series around a pole, it's radius of convergence will be the distance to the next closest place the function is not analytic.Your final example is true everywhere that the expressions are defined. $$frac 1{1-z} = sum z^n$$ is true where the series converges, which includes $|z| < 1$ and excludes $|z| > 1$. But $$frac{-1}zfrac 1{1-z}=frac{-1}z sum z^n$$ also requires $frac 1z$ to exist, so excludes $z = 0$.
    $endgroup$
    – Paul Sinclair
    Dec 1 '18 at 5:50














0












0








0





$begingroup$


I just wanna see if I have some of the fundamentals nailed in terms of understanding.



Say we have some complex valued function defined as



$$f(z)=tfrac{1}{z(z-1)}$$



and we want to evaluate it's laurent series centred at $z_0=0$ on the annulus $0<|z|<1$.



Or take the function



$$f(z)=tfrac{1}{(z-1)(z+1)}$$



this time taking it's Laurent series centred at $z_0=1$ on the annulus $0<|z-1|<2$



I have a few questions regarding said laurent series:



i) my first question is just on the general situation weve got here . I believe in the first example I mentioned that we have a function which has two singularities, a simple pole at zero and another at 1. Then we have defined a region on which to calculate the laurent series that stops us from hitting the singularities by only calculating at a distance greater to one and less than the other . similarly we have in my example a complex valued function defined on some region but this time it's singularities are +-1 . and we want to centre our expansion at 1 so no we define our region in such a way that the distance between 1( which we'll never hit because we require the distance between it and any point to be greater than zero ) and any other point is less than 2 so we know have a circle that is centred at one and has one side touching 3 and the other -1 (it's singularity) but never touches either.Is this correct ?



ii)a. when actually calculating the series mentioned in my first series I know we evaluate in a region valid for $|z|<1$ in the following way :



$$tfrac{1}{z(z-1)}=tfrac{-1}{z}tfrac{1}{1-z}=tfrac{-1}{z} Sigma_{n=0}z^n$$ which is valid for $|z|<1/|c|$ c in this case being the coeffiecient of z in in the secon equality above.



My question however is this, evaluating for $|z|<1$ is the same as evaluating for $0<|z|<1$ in this case right ? , because the series i found is valid below 1 up until 0 which it is undefined, at right ?










share|cite|improve this question











$endgroup$




I just wanna see if I have some of the fundamentals nailed in terms of understanding.



Say we have some complex valued function defined as



$$f(z)=tfrac{1}{z(z-1)}$$



and we want to evaluate it's laurent series centred at $z_0=0$ on the annulus $0<|z|<1$.



Or take the function



$$f(z)=tfrac{1}{(z-1)(z+1)}$$



this time taking it's Laurent series centred at $z_0=1$ on the annulus $0<|z-1|<2$



I have a few questions regarding said laurent series:



i) my first question is just on the general situation weve got here . I believe in the first example I mentioned that we have a function which has two singularities, a simple pole at zero and another at 1. Then we have defined a region on which to calculate the laurent series that stops us from hitting the singularities by only calculating at a distance greater to one and less than the other . similarly we have in my example a complex valued function defined on some region but this time it's singularities are +-1 . and we want to centre our expansion at 1 so no we define our region in such a way that the distance between 1( which we'll never hit because we require the distance between it and any point to be greater than zero ) and any other point is less than 2 so we know have a circle that is centred at one and has one side touching 3 and the other -1 (it's singularity) but never touches either.Is this correct ?



ii)a. when actually calculating the series mentioned in my first series I know we evaluate in a region valid for $|z|<1$ in the following way :



$$tfrac{1}{z(z-1)}=tfrac{-1}{z}tfrac{1}{1-z}=tfrac{-1}{z} Sigma_{n=0}z^n$$ which is valid for $|z|<1/|c|$ c in this case being the coeffiecient of z in in the secon equality above.



My question however is this, evaluating for $|z|<1$ is the same as evaluating for $0<|z|<1$ in this case right ? , because the series i found is valid below 1 up until 0 which it is undefined, at right ?







complex-analysis laurent-series






share|cite|improve this question















share|cite|improve this question













share|cite|improve this question




share|cite|improve this question








edited Nov 30 '18 at 19:13







can'tcauchy

















asked Nov 30 '18 at 18:37









can'tcauchycan'tcauchy

999417




999417












  • $begingroup$
    Your typography is a little difficult for me to read, but at least for your first question, I think you have the right idea. The region you define the Laurent series on won't contain a singularity, but the singularities can be on the boundary of the region (in fact, regions of convergence for Laurent series are either disks or annuli, depending on the singularities).
    $endgroup$
    – Clayton
    Nov 30 '18 at 18:55






  • 1




    $begingroup$
    We actually don't choose the region. The Laurent series itself does that. If you take the Laurent series around a pole, it's radius of convergence will be the distance to the next closest place the function is not analytic.Your final example is true everywhere that the expressions are defined. $$frac 1{1-z} = sum z^n$$ is true where the series converges, which includes $|z| < 1$ and excludes $|z| > 1$. But $$frac{-1}zfrac 1{1-z}=frac{-1}z sum z^n$$ also requires $frac 1z$ to exist, so excludes $z = 0$.
    $endgroup$
    – Paul Sinclair
    Dec 1 '18 at 5:50


















  • $begingroup$
    Your typography is a little difficult for me to read, but at least for your first question, I think you have the right idea. The region you define the Laurent series on won't contain a singularity, but the singularities can be on the boundary of the region (in fact, regions of convergence for Laurent series are either disks or annuli, depending on the singularities).
    $endgroup$
    – Clayton
    Nov 30 '18 at 18:55






  • 1




    $begingroup$
    We actually don't choose the region. The Laurent series itself does that. If you take the Laurent series around a pole, it's radius of convergence will be the distance to the next closest place the function is not analytic.Your final example is true everywhere that the expressions are defined. $$frac 1{1-z} = sum z^n$$ is true where the series converges, which includes $|z| < 1$ and excludes $|z| > 1$. But $$frac{-1}zfrac 1{1-z}=frac{-1}z sum z^n$$ also requires $frac 1z$ to exist, so excludes $z = 0$.
    $endgroup$
    – Paul Sinclair
    Dec 1 '18 at 5:50
















$begingroup$
Your typography is a little difficult for me to read, but at least for your first question, I think you have the right idea. The region you define the Laurent series on won't contain a singularity, but the singularities can be on the boundary of the region (in fact, regions of convergence for Laurent series are either disks or annuli, depending on the singularities).
$endgroup$
– Clayton
Nov 30 '18 at 18:55




$begingroup$
Your typography is a little difficult for me to read, but at least for your first question, I think you have the right idea. The region you define the Laurent series on won't contain a singularity, but the singularities can be on the boundary of the region (in fact, regions of convergence for Laurent series are either disks or annuli, depending on the singularities).
$endgroup$
– Clayton
Nov 30 '18 at 18:55




1




1




$begingroup$
We actually don't choose the region. The Laurent series itself does that. If you take the Laurent series around a pole, it's radius of convergence will be the distance to the next closest place the function is not analytic.Your final example is true everywhere that the expressions are defined. $$frac 1{1-z} = sum z^n$$ is true where the series converges, which includes $|z| < 1$ and excludes $|z| > 1$. But $$frac{-1}zfrac 1{1-z}=frac{-1}z sum z^n$$ also requires $frac 1z$ to exist, so excludes $z = 0$.
$endgroup$
– Paul Sinclair
Dec 1 '18 at 5:50




$begingroup$
We actually don't choose the region. The Laurent series itself does that. If you take the Laurent series around a pole, it's radius of convergence will be the distance to the next closest place the function is not analytic.Your final example is true everywhere that the expressions are defined. $$frac 1{1-z} = sum z^n$$ is true where the series converges, which includes $|z| < 1$ and excludes $|z| > 1$. But $$frac{-1}zfrac 1{1-z}=frac{-1}z sum z^n$$ also requires $frac 1z$ to exist, so excludes $z = 0$.
$endgroup$
– Paul Sinclair
Dec 1 '18 at 5:50










0






active

oldest

votes











Your Answer





StackExchange.ifUsing("editor", function () {
return StackExchange.using("mathjaxEditing", function () {
StackExchange.MarkdownEditor.creationCallbacks.add(function (editor, postfix) {
StackExchange.mathjaxEditing.prepareWmdForMathJax(editor, postfix, [["$", "$"], ["\\(","\\)"]]);
});
});
}, "mathjax-editing");

StackExchange.ready(function() {
var channelOptions = {
tags: "".split(" "),
id: "69"
};
initTagRenderer("".split(" "), "".split(" "), channelOptions);

StackExchange.using("externalEditor", function() {
// Have to fire editor after snippets, if snippets enabled
if (StackExchange.settings.snippets.snippetsEnabled) {
StackExchange.using("snippets", function() {
createEditor();
});
}
else {
createEditor();
}
});

function createEditor() {
StackExchange.prepareEditor({
heartbeatType: 'answer',
autoActivateHeartbeat: false,
convertImagesToLinks: true,
noModals: true,
showLowRepImageUploadWarning: true,
reputationToPostImages: 10,
bindNavPrevention: true,
postfix: "",
imageUploader: {
brandingHtml: "Powered by u003ca class="icon-imgur-white" href="https://imgur.com/"u003eu003c/au003e",
contentPolicyHtml: "User contributions licensed under u003ca href="https://creativecommons.org/licenses/by-sa/3.0/"u003ecc by-sa 3.0 with attribution requiredu003c/au003e u003ca href="https://stackoverflow.com/legal/content-policy"u003e(content policy)u003c/au003e",
allowUrls: true
},
noCode: true, onDemand: true,
discardSelector: ".discard-answer"
,immediatelyShowMarkdownHelp:true
});


}
});














draft saved

draft discarded


















StackExchange.ready(
function () {
StackExchange.openid.initPostLogin('.new-post-login', 'https%3a%2f%2fmath.stackexchange.com%2fquestions%2f3020470%2fhelp-understanding-the-laurent-series%23new-answer', 'question_page');
}
);

Post as a guest















Required, but never shown

























0






active

oldest

votes








0






active

oldest

votes









active

oldest

votes






active

oldest

votes
















draft saved

draft discarded




















































Thanks for contributing an answer to Mathematics Stack Exchange!


  • Please be sure to answer the question. Provide details and share your research!

But avoid



  • Asking for help, clarification, or responding to other answers.

  • Making statements based on opinion; back them up with references or personal experience.


Use MathJax to format equations. MathJax reference.


To learn more, see our tips on writing great answers.




draft saved


draft discarded














StackExchange.ready(
function () {
StackExchange.openid.initPostLogin('.new-post-login', 'https%3a%2f%2fmath.stackexchange.com%2fquestions%2f3020470%2fhelp-understanding-the-laurent-series%23new-answer', 'question_page');
}
);

Post as a guest















Required, but never shown





















































Required, but never shown














Required, but never shown












Required, but never shown







Required, but never shown

































Required, but never shown














Required, but never shown












Required, but never shown







Required, but never shown







Popular posts from this blog

Quarter-circle Tiles

build a pushdown automaton that recognizes the reverse language of a given pushdown automaton?

Mont Emei